As it is presented in the passage, the approach to history taken by mainstream U.S. historians of the late nineteenth...

Ryan on May 16, 2019

Example 1 Question 7

In the example at time 16:00, C is dismissed where G is placed in the 4th time slot because that still left 2 slots open for LO. However if G is fourth, and P is placed 7th, placing LO in the 5/6th spot would not work because N must come after L and O. So then the only spot where L and O can go is 2nd and 3rd. In the explanation this is not gone over and the answer chosen is D which also seems correct to me

Replies
Create a free account to read and take part in forum discussions.

Already have an account? log in

Ravi on May 17, 2019

@RyanCamp,

Happy to help.

The problem with (C) is that if we put G 4th, then P can go in 1 or 7.
This means that LO can go in 2 and 3 or 5 and 6, so O's position is
not completely determined if G is put in 4th. This is why (C) is
incorrect.

Does that make sense? Let us know if you have any other questions!

Christopher on September 27, 2019

This makes sense but what you said should be commented on. Since during testing I would run both models to assume correctness.